Hattie had $3,000 to invest and wants to earn 10.6% interest per year. She will put some of the money into an account that earns 12% per year and the rest into an account that earns 10% per year. How much money should she put into each account?

Hattie Had $3,000 To Invest And Wants To Earn 10.6% Interest Per Year. She Will Put Some Of The Money

Answers

Answer 1

Answer:

900 at 12%

2100 at 10%

Step-by-step explanation:

Let x= amount invested at 12%

let y= amount invested at 10%

with that being said we can write the two equation

Equation 1: x+y=3000

Equation 2: 3000*.106=.12x+.1y

isolte x from equation 1

x= 3000-y

plug this into equation 2

318=.12(3000-y)+.1y

318=360-.12y+.1y

-42= -.02y

y= 2100

Plug this into equation 1

x+2100=3000

x=900

Answer 2

she should invest $900 into the account earning 12% interest and $2100 into the account earning 10% interest.

How to determine How much money should she put into each account

Let's denote the amount of money Hattie invests at 12% as \(x\) dollars, and the amount she invests at 10% as \(\$3000 - x\) dollars.

The formula for calculating interest is: \(\text{Interest} = \text{Principal} \times \text{Rate} \times \text{Time}\).

For the 12% account:

Interest_12% = \(x \times 0.12 \times 1\) (1 year)

For the 10% account:

Interest_10% = \((3000 - x) \times 0.10 \times 1\) (1 year)

Hattie wants to earn 10.6% interest on the total investment, so we can set up the equation:

\(\text{Total Interest} = \text{Interest}_12% + \text{Interest}_10%\)

\(3000 \times 0.106 = x \times 0.12 + (3000 - x) \times 0.10\)

Now, solve for \(x\):

\(318 = 0.12x + 300 - 0.10x\)

\(318 = 0.02x + 300\)

\(18 = 0.02x\)

\(x = 900\)

Hattie should invest $900 at 12% and \(3000 - 900 = 2100\) at 10%.

Therefore, she should invest $900 into the account earning 12% interest and $2100 into the account earning 10% interest.

Learn more about interest at https://brainly.com/question/29451175

#SPJ3


Related Questions

A seller of the property listed at $200,000 excepted a 90% offer the home appraised at $185,000 and the buyers obtained a loan for 85% for 30 years at 5% interest what is the first months interest

Answers

Answer:

$637.50

Step-by-step explanation:

According to the Question,

Given That, A seller of the property listed at $200,000 excepted a 90% offer the home appraised at $185,000 and the buyers obtained a loan for 85% for 30 years at 5% interest

Thus, the first months interest is

$200,000 list price x 0.90 = $180,000 contract sales price.

Since lender always uses the less of the appraised value or the contract sales price, use $180,00 for the remainder of the calculations.

$180,000 contract sales price x 0.85 LTV = $153,000 loan. $153,000 loan x 0.05 interest rate = $7,650 annual interest. $7,650 ÷ 12 = $637.50 monthly interest payment for the first month.

Answer:

$637.50

Step-by-step explanation:

The appraised value is irrelevant. The lender will consider the lower of the appraised value or the agreed purchase price.

The term of the loan is also irrelevant. It is not an amortization problem.

The first month’s interest is $637.50.

The weights for newborn babies is approximately normally distributed with a mean of 5.4 pounds and a standard deviation of 1.8 pounds. Consider a group of 1500 newborn babies: 1. How many would you expect to weigh between 3 and 6 pounds

Answers

Answer:

You would expect 807 babies  to weigh between 3 and 6 pounds.

Step-by-step explanation:

We are given that

Mean,[tex]\mu=5.4[/tex]pounds

Standard deviation,[tex]\sigma=1.8[/tex]pounds

n=1500

We have to find how  many would you expect to weigh between 3 and 6 pounds.

The weights for newborn babies is approximately normally distributed.

Now,

[tex]P(3<x<6)=P(\frac{3-5.4}{1.8}<\frac{x-\mu}{\sigma}<\frac{6-5.4}{1.8})[/tex]

[tex]=P(-1.33<Z<0.33)[/tex]

[tex]P(3<x<6)=P(Z<0.33)-P(Z<-1.33)[/tex]

[tex]P(3<x<6)=0.62930-0.09176[/tex]

[tex]P(3<x<6)=0.538[/tex]

Number of newborn  babies expect to weigh between 3 and 6 pounds

=[tex]1500\times 0.538=807[/tex]

(3x^3)^2 write without exponent

Answers

Answer:

9*x*x*x*x*x*x.

Step-by-step explanation:

(3x^3)^2

= 3^2 * x^(3*2)

= 3^2 * x^6

= 9*x*x*x*x*x*x

Write the number 52 as a product of primes. SHOW ALL WORK

Answers

AnsweStep-by-step explanation:

lol

Use the arithmetic progression formula to find the sum of integers from 75 to 100.75,76,77....99,100.​

Answers

Answer:

The sum is 2275

Step-by-step explanation:

Given

[tex]75,76,77....99,100[/tex]

Required

The sum

Using arithmetic progression, we have:

[tex]S_n = \frac{n}{2}(T_1 + T_n)[/tex]

Where:

[tex]T_1 = 75[/tex] --- first term

[tex]T_n = 100[/tex] --- last term

[tex]n = T_n - T_1 + 1[/tex]

[tex]n = 100 - 75 + 1 = 26[/tex]

So, we have:

[tex]S_n = \frac{n}{2}(T_1 + T_n)[/tex]

[tex]S_n = \frac{26}{2}*(75 + 100)[/tex]

[tex]S_n = 13*175[/tex]

[tex]S_n = 2275[/tex]

Brian made $198 for 11 hours of work.
At the same rate, how many hours would he have to work to make $324 ?

Answers

Answer:

18 hours

Step-by-step explanation:

Forst u must find out how much u get for 1 hour which is 198/11=18 so every hour u get 18 dollars.

Next, Ypu must divide 324/18 to see how many hours he worked which we get 18

Finally 18 is the answer!

Step-by-step explanation:

If he made $198 in 11hrs

how many hours will he take to make $324

Let hours be x

$198=11hours

$324= x hours

= $324 *11hours / $198

= 18 hours

I hope this helps.

Addison drove 960 miles in 16 hours what was her speed in miles per hour

Answers

The correct answer is 60 miles per hour

You would divide 960 by 16 and will get 60 as your answer

Answer:

60 miles/hour

Step-by-step explanation:

960÷16

=60 hours

Sally bought five books.Their mean price was 3.25. The total cost for four books was 11.75.what was the cost of the fifth book​

Answers

Answer:

$4.50

Step-by-step explanation:

Use the mean formula: mean = sum of elements / number of elements

Let x represent the cost of the fifth book, and solve for x:

mean = sum of elements / number of elements

3.25 = (11.75 + x) / 5

16.25 = 11.75 + x

4.5 = x

So, the cost of the fifth book was $4.50

Find the equation of the lines in problem 1 (0,0) slope =2.

Answers

Y= 2x as simple as that

Answer:

y = 2x

Step-by-step explanation:

Given that , the line passes through the point (0,0) and has a slope of 2. So here we can use the point slope form of the line as ,

[tex]\implies y- y_1 = m( x - x_1) \\\\\implies y - 0 = 2( x - 0 ) \\\\\implies y = 2(x) \\\\\implies \underline{\underline{y = 2x }}[/tex]

A cell site is a site where electronic communications equipment is placed in a cellular network for the use of mobile phones. The numbers y of cell sites from 1985 through 2011 can be modeled byy = 269573/1+985e^-0.308t where t represents the year, with t = 5 corresponding to 1985. Use the model to find the numbers of cell sites in the years 1998, 2003, and 2006.

Answers

Answer:

(a) 3178

(b) 14231

(c) 33152

Step-by-step explanation:

Given

[tex]y = \frac{269573}{1+985e^{-0.308t}}[/tex]

Solving (a): Year = 1998

1998 means t = 8 i.e. 1998 - 1990

So:

[tex]y = \frac{269573}{1+985e^{-0.308*8}}[/tex]

[tex]y = \frac{269573}{1+985e^{-2.464}}[/tex]

[tex]y = \frac{269573}{1+985*0.08509}[/tex]

[tex]y = \frac{269573}{84.81365}[/tex]

[tex]y = 3178[/tex] --- approximated

Solving (b): Year = 2003

2003 means t = 13 i.e. 2003 - 1990

So:

[tex]y = \frac{269573}{1+985e^{-0.308*13}}[/tex]

[tex]y = \frac{269573}{1+985e^{-4.004}}[/tex]

[tex]y = \frac{269573}{1+985*0.01824}[/tex]

[tex]y = \frac{269573}{18.9664}[/tex]

[tex]y = 14213[/tex] --- approximated

Solving (c): Year = 2006

2006 means t = 16 i.e. 2006 - 1990

So:

[tex]y = \frac{269573}{1+985e^{-0.308*16}}[/tex]

[tex]y = \frac{269573}{1+985e^{-4.928}}[/tex]

[tex]y = \frac{269573}{1+985*0.00724}[/tex]

[tex]y = \frac{269573}{8.1314}[/tex]

[tex]y = 33152[/tex] --- approximated

A loan of £1000 has a compound interest rate of 2.7% charged monthly. Express the original loan as a percentage of the total amount awed after 2 months if no payment are made

Answers

Answer:

£1054.729

Step-by-step explanation:

To find compound interest you need to use the equation 1000(1.027)^x.

To find the interest rate (1.027):

100 + 2.7 = 102.7

102.7 / 100 = 1.027

The value of x is the amount of months if no payment is made in this situation, so 2 would be the x value for this problem.

Hope this helps!  

Tonya wants to estimate what proportion of the students in her dormitory like the dorm food. She interviews a simple random sample of 50 students living in the dormitory. She finds that 14 think the dorm food is good. Find a 90% confidence interval for the true proportion of students that think the dorm food is good.
a. 0.176 to 0.384
b. 28%
c. 0.28 +/- 0.03
d. 0.156 to 0.404

Answers

Answer:

Step-by-step explanation:

The solution of the problem has been solved on paper and attached in the attachment section. Kindly refer to that and feel free to ask any doubt.

I need help asap will give five stars and mark u the brainliest

Answers

Answer:

-00

OKAY

OKAY

OK

OK

DGSOTSITS6EUEYEYTITUTUTIT

Which expression is equivalent to the following complex fraction?

Answers

Step-by-step explanation:

Option B is correct. Refer to the attachment!

A bank gives you a loan of 1,500,000 Baht to buy a house. The interest rate of the loan is 0.01% per day (Using 1 year = 365 days)

How much interest you pay after 10 years

Answers

Answer:

15.000 is cost so relate it with 265..hope it is help u.

Xét mô hình thu nhập quốc dân hai thành phần sau đây

dY/dt= 0.5(C + I – Y)
C = 0.6Y + 600
I = 0.2Y + 400.

Tìm biểu diễn của Y(t) với Y(0) = 9000. Mô hình này ổn định hay không ổn định?

Answers

Answer:

No se me puedes ayudar por fa

PLEASE HELP!!!! Choose the best graph that represents the linear equation: 4x + y = 0 Graph A On a coordinate plane, a line goes through (0, 0) and (1, 4). Graph B On a coordinate plane, a line goes through (0, 0) and (1, negative 4). Graph C On a coordinate plane, a line goes through (0, 0) and (4, negative 1). Graph D On a coordinate plane, a line goes through (0, 0) and (4, 1). a. Graph A c. Graph C b. Graph B d. Graph D Please select the best answer from the choices provided A B C D

Answers

Answer:

see graph...

B On a coordinate plane, a line goes through (0, 0) and (1, negative 4).

Step-by-step explanation:

Answer:

Step-by-step explanation:

To determine the graph that represents the linear equation 4x + y = 0, we need to rearrange the equation in slope-intercept form (y = mx + b), where m represents the slope and b represents the y-intercept.

Starting with the given equation:

4x + y = 0

Subtracting 4x from both sides:

y = -4x

Now we have the equation in the form y = mx + b, where m = -4 (the coefficient of x) and b = 0.

Looking at the given graphs:

Graph A: The line goes through (0, 0) and (1, 4). This graph does not represent the equation y = -4x, as the slope is 4, not -4.

Graph B: The line goes through (0, 0) and (1, -4). This graph does represent the equation y = -4x, as it has a slope of -4 and passes through the origin.

Graph C: The line goes through (0, 0) and (4, -1). This graph does not represent the equation y = -4x, as it does not have the correct slope.

Graph D: The line goes through (0, 0) and (4, 1). This graph does not represent the equation y = -4x, as it does not have the correct slope.

Therefore, the best graph that represents the linear equation 4x + y = 0 is:

b. Graph B

three friends, akira,bruno and carmela pooled thier money to start a lemonade stand. akria contributes $25, bruno contributed $20 and carmela contributed $35. after a month, thier lemoneade stand had earned 2000, and they want to distribute this money in the same ratio as the money that was invested. how many dollars will brouno recieve
plz explian

Answers

9514 1404 393

Answer:

  $500

Step-by-step explanation:

Bruno's fraction of the total contribution was ...

  Bruno / Total = $20/($25 +20 +35) = 20/80 = 1/4

Then Bruno's share of the earnings is this same fraction, so is ...

  (1/4) × ($2000) = $500

F(x)=-x^2-4 for x= -3

Answers

Answer:

5

Step-by-step explanation:

Given:

f(x)=-x²-4

Substitute x= -3:

f(-3) = (-3)² - 4 = 9 - 4 = 5

Determine if each proportion on the left is true or false

Answers

9514 1404 393

Answer:

FalseTrueTrueTrue

Step-by-step explanation:

An easy way to tell if the equation is true is to multiply it by the product of the denominators (cross multiply).

  7×7 ≠ 12×4 . . . . proportion is False

  9×8 = 36×2 . . . . proportion is True

  8×9 = 24×3 . . . . proportion is True

  5×104 = 8×65 . . . . proportion is True

Find the probability that when a couple has ​children, at least one of them is a . ​(Assume that boys and girls are equally​ likely.)

Answers

Answer:

[tex]P(At\ least\ one\ girl) = 0.875[/tex]

Step-by-step explanation:

Given

[tex]n = 3[/tex]

[tex]B \to boys[/tex]

[tex]G \to girls[/tex]

[tex]P(G) = P(B) = 0.5[/tex] --- equal probability

See comment for complete question

Required:

[tex]P(At\ least\ one\ girl)[/tex]

To do this, we make use of complement rule:

[tex]P(At\ least\ one\ girl) = 1 - P(No\ girl)[/tex]

The event that there is no girl out of the 3 children is: B B B

And the probability is:

[tex]P(No\ Girl) = P(B) * P(B) * P(B)[/tex]

[tex]P(No\ Girl) = 0.5*0.5*0.5[/tex]

[tex]P(No\ Girl) = 0.125[/tex]

So:

[tex]P(At\ least\ one\ girl) = 1 - P(No\ girl)[/tex]

[tex]P(At\ least\ one\ girl) = 1 - 0.125[/tex]

[tex]P(At\ least\ one\ girl) = 0.875[/tex]

a circle has a radius that is 4 centimeters long. if a central angle has a measure of 3 radiants, what is the length of the arc that corresponds to the angle ?

A. 12 centimeters
B. 4 centimeters
C. 3 centimeters
D. 7 centimeters

Answers

Answer:

A number is right 12 centimetres

What is the prime factorization of 30?
O A. 2.2.3.5
O B. 5.6
O C. 3.10
O D. 2.3.5

Answers

D. 2.3.5 is the correct answer

translate to a system of equations but do not solve.

A non-toxic floor wax can be made from lemon juice and food grade linseed oil. The amount of oil should be twice the amount of lemon juice. How much of each ingredient is needed to make 30 oz of floor wax?

let x represent the number of ounces of lemon juice and y represent the number of ounces of linseed oil.
complete the system of equations.

y =
x+y =​

Answers

Answer:

x + y = 30

y = 2x

Step-by-step explanation:

x = number of ounces of lemon juice

y = number of ounces of linseed oil

How much of each ingredient is needed to make 30 oz of floor wax?

x + y = 30

The amount of oil should be twice the amount of lemon juice.

y = 2x

Answer:

x + y = 30

y = 2x

use the figure below to find the answer. find y.

Answers

9514 1404 393

Answer:

  y = 7√2

Step-by-step explanation:

We are given the side opposite the angle, and we want to find the hypotenuse. The relevant trig relation is ...

  Sin = Opposite/Hypotenuse

  sin(45°) = 7/y

  y = 7/sin(45°) = 7/(1/√2)

  y = 7√2

__

Additional comment

In this 45°-45°-90° "special" right triangle, the two legs are the same length. Thus, ...

  x = 7

A coordinate plane with a line passing through the points (0, negative 1, 0) and (4, 0).
What is the equation of the graphed line written in standard form?

x – 4y = 4
x + 4y = 4
y = y equals StartFraction one-fourth EndFraction x minus 1.x – 1
y = –y equals negative StartFraction one-fourth EndFraction x minus 1.x – 1

Answers

Answer:

x-4y=4

Step-by-step explanation:

The equation of a straight line is in slope intercept form:

y = mx + b;

where y and x are variables, m is the slope of the line and b is the y intercept.

The standard form of a line is:

Ax + By = C

where A, B and C are constants, x and y are variables

The equation of a straight line passing through the points (x₁, y₁) and (x₂, y₂) is given by:

[tex]y-y_1=\frac{y_2-y_1}{x_2-x_1}(x-x_1)\\[/tex]

Given that a line passes through the points (0, -1) and (4, 0), the equation of the line is given by:

[tex]y-(-1)=\frac{0-(-1)}{4-0} (x-0)\\\\y+1=\frac{1}{4} x\\\\y=\frac{1}{4} x-1\\\\4y=x-4\\\\x-4y=4[/tex]

Answer:

Step-by-step explanation:

the difference when a number doubled is subtracted from 3​

Answers

Answer: Let the number be x

3-2x is the equation.

A car and a motorcycle whose average rates are in the ratio of 4:5 travel a distance of 160 miles. If the motorcycle
travels 1/2 hour less than the car, find the average rate of each.

Answers

Answer:

Step-by-step explanation:

I always advise my students to make a table of information for these story problems because trying to keep track of the information otherwise is a nightmare. The table will look like this:

           d      =      r      *      t

m

c

m is motorcycle and c is car.

First thing we are told is that the ratio of m's speed to c's speed is 5:4; that means that we can divide 5/4 to find out how many times faster m is going than c.

5/4 = 1.25 so we have a couple of values to put into the table right away, along with the fact that they are both traveling the same distance of 160 miles.

             d      =     r      *      t

m        160     =   1.25r

c         160     =     r

The last thing we have to fill in is the time. If m travels a half hour less than c, c is driving a half hour more than m, right? Filling that in:

            d      =      r      *      t

m       160     =   1.25r   *     t

c        160     =      r       *  t + .5

Now we have our 2 equations. Looking at the top row of the table gives us the formula we need to solve this problem. It tells us, in other words, what we are going to be doing with these columns of numbers. Distance equals the rate times the time. For the motorcycle, the equation is:

160 = (1.25r)t  and that seems pretty useless since we still have 2 unknowns in there and you can only have 1 unknown in 1 equation. Let's see what the equation for the car is.

160 = (t + .5)r  Same problem.

Let's go back to the equation for the motorcycle and since we are looking for the rates of each, let's solve that equation for time in terms of rate (solve it for t):

[tex]t=\frac{160}{1.25r}[/tex] and sub that into the car's equation in place of t:

[tex]160=r(\frac{160}{1.25r})+.5r[/tex] and simplify. The r's to the left of the plus sign cancel out leaving us with:

[tex]160=(\frac{160}{1.25})+.5r[/tex] and divide those numbers inside the parenthesis to get:

160 = 128 + .5r and subtract 128 from both sides to get:

32 = .5r and finally divide by .5 to get

r = 64 miles/hour

The car goes 64 mph and the motorcycle goes 1.25 times that so,

m = 1.25(64) and

m = 80 mph

if sine Theta is less than 0 and tan Theta is greater than 0 then​

Answers

Answer:

Sine Theta is a negative number, Tan Theta is a greater number then zero.

Step-by-step explanation:

If Sine Theta is less then zero, she is a negative number. So 0 - y = -y.

So if Tan Theta is a greater number than zero, her number is not negative. So 0 + y = y

I hope this helped! I didn’t really understand the question though.

find the mean proportional between 9 and 1/36​

Answers

Answer:

The mean proportional between 1/36 and 1/9 would be 1/18.

Other Questions
Four pounds of tea and 5 pounds of coffee cost $88.30, while 9 pounds of tea andsix pounds of coffee cost $144.60. What is the cost of each per pound?Cost of 1 pound of tea = $4.78 and cost of 1 pound of coffee = $12.88Cost of 1 pound of tea = $9.20 and cost of 1 pound of coffee = $10.30O Cost of 1 pound of tea = $8.20 and cost of 1 pound of coffee = $10.30O Cost of 1 pound of tea = $9.20 and cost of 1 pound of coffee = $9.30 What is the first step in the curved arrow mechanism for the chlorination of benzene in the presence of FeCl3 what is carrageenan ? en el diagrama de venn donde van ubicados estos numeros?0,88888....1 sobre 7 pi-6 sobre 3 4E 55 sobre 056 sobre 9 -0,65999999 round 6.8 to nearest hundredth This one too, please?(3rd time)I need explanation for your answers, even though it's multiple choices, I still need your explanation for it.DUE TOMORROW!If your answer is NONSENSE it will be deleted as soon as possible!But if your answer is CORRECT, HELPFUL, HAS AN EXPLANATION, I'll chose your answer as the BRAINLIEST ANSWER! Someone please help me ASAP! A modified roulette wheel has 36 slots. One slot is 0, another is 00, and the others are numbered 1 through 34, respectively. You are placing a bet that the outcome is an even number. (In roulette, 0 and 00 are neither odd nor even.) a. What is your probability of winning? An 8-oz bottle of hair spray costs $4.46. Find the unit price in cents per ounce Black Acres Apartment, Inc needs to compute taxable income (TI) for the preceding year and wants your assistance. The effective gross income (EGI) was $52,000; operating expenses were $19,000; $2,000 was put into a fund for future replacement of stoves and refrigerators; debt service was $26,662, of which $25,126 was interest; and the deprecation deduction was $17,000. Compute the taxable income from operations: Complete the remainder of the table for the given function rule: y= x/4 + 7 I need help ASAP!List the fractions and decimals in order least to greatest.Question1. 2.3 , 2 4/5 , 2.6 Question 2. 0.5 , 3/16 , 0.75 , 5/48 Who ever helped me thank you :D verify that whether -2 and 3 are zeroes of the polynomial x^2-x=6PLEASE HELP what did sulla do to recognize the government structure of romeA he disbanded the senateB he established an emperorC he eliminated one of the consulsD he removed all power from the tribunes Which statement best describes the rhetorical technique used in this excerpt? A 6 paragraph essay on anything you can do I can do better What type of business would publish a new product marketing promotion in the local newspaper, online, and in stores Which person is an example of an entrepreneur?A - a lawyer who represents clients with very little money.B - a restaurant server who take college classes after work?C - an architect who designs buildings for the government D - an inventor who creates a new computer company Which conclusion can be drawn from this chart? You offer to sell a used car for $1,895. Yesterday you purchased the car for $1,755. What percentage markup on cost are you charging (to the nearest tenth)?